¿Cómo se puede explicar esta aparente contradicción entre la conservación de la energía y la dependencia del marco de KE?

Partiendo de las premisas que brotan constantes k y el desplazamiento, y por lo tanto la energía elástica almacenada en un resorte son cantidades invariantes de marco, aquí hay un experimento mental que lleva a la pregunta.

Un resorte comprimido que contiene Q julios de energía elástica descansa sobre una superficie horizontal sin fricción con su eje paralelo a la superficie y un extremo fijo a la superficie. Un bloque de material se pone en contacto con el otro extremo. Se suelta el resorte y un observador estacionario concluye correctamente que, dado que Q julios de energía elástica se han transferido al bloque, el bloque ahora tiene Q julios de KE.

Un segundo observador, moviéndose en la misma dirección que el bloque con velocidad v que resulta ser igual a la velocidad final del bloque en el marco estacionario, está de acuerdo en que se transfirieron Q julios de energía al bloque desde el resorte como energía cinética, pero observa que el resultado fue que el bloque se detuvo y tiene cero EC.

Eso solo funciona matemáticamente si el segundo observador calculó la KE inicial como negativa. Pero KE nunca es realmente negativo, por lo que parece que las matemáticas asociadas con la dependencia del marco de KE no describen la realidad cuando una forma de energía invariante de marco, que no es KE, se transfiere a un objeto como KE.

¿Cómo se puede explicar esta aparente contradicción entre la conservación de la energía y la dependencia del marco de KE?

Defina en qué medida la energía cinética y la energía cinética medida son cantidades diferentes. Además, si hablas de invariancia de marco, sería bueno saber de qué grupo de simetría estás hablando, solo para estar seguro...
@Sanya Parece que está listo para responder la pregunta basándose solo en el encabezado. Digo esto porque la diferencia entre KE real y medida es bastante clara en el cuerpo de la pregunta.
Leí todo tu mensaje. No hay una definición concisa allí que pueda entender.
Resalte, tal vez en negrita, la parte exacta que cree que es una contradicción. De lo contrario, es difícil ver cuál es su argumento.
@knzhou Eso se ha hecho. Gracias por la sugerencia.
@knzhou No traté de incluir esto en la pregunta, pero también es una contradicción creer que la energía eléctrica en una celda alcalina y la energía elástica en un resorte son invariantes mientras que la energía cinética no lo es, a menos que creas que la energía cinética es una entidad física diferente a las demás.
Veo que a alguien no le gusta la pregunta e hizo uno de esos votos anónimos sin decir por qué. ¿Es esta realmente una pregunta de baja calidad, irreflexiva o fuera de tema? ¿En realidad? ¿O se metió debajo de la piel de alguien por una razón más profunda?
-1 de mí. Perdón por la demora en explicar. Me parece que está contrastando (i) la conservación de la energía (en la que, como dice, todos los observadores están de acuerdo), con (ii) la KE final medida por diferentes observadores. Creo que las preguntas relacionadas ya han respondido esta distinción: diferentes observadores pueden estar en desacuerdo sobre cuánto KE tiene un objeto, pero todos están de acuerdo en que la energía se conserva. Sería útil si citara respuestas que afirma haber evitado el problema.
@sammy gerbil Gracias por intensificar y aclarar su posición. Pero lo que distingue mi pregunta de la que usted llama un posible duplicado es la transferencia de una cantidad de energía, acordada por ambos observadores, de una forma distinta a la energía cinética a la energía cinética.
@sammygerbil Entonces, ¿votó en contra porque el OP no entiende lo que está preguntando ...? Eso hace que sea bastante difícil hacer preguntas, ¿no? Esta pregunta está bien investigada y bien explicada y él hace un esfuerzo por explicar cómo esto no se ha respondido ya en otra parte. No veo razones para votar negativo
@Steeven: No, voté en contra porque creo que la pregunta no es útil, ya que es un duplicado poco claro de preguntas anteriores. No hay ninguna cita de otras preguntas y la distinción reclamada no me queda clara. Sin embargo, no he votado para cerrar.
@D.Ennis: Lo siento, todavía no entiendo la distinción que está haciendo entre su pregunta y la que cité. Está preguntando si 'KE' es invariante mientras que 'KE medido' no lo es. Al igual que Sanya, no entiendo la diferencia entre estos dos términos. Si crees que hay uno, por favor, ¿podrías definir cuál es? ... En su ejemplo, parece estar preguntando "¿Dónde está la energía que falta?" Las preguntas relacionadas ya han abordado este problema, al igual que la respuesta de MDC.
@sammygerbil En la redacción de mi pregunta pretendo que "medición de KE" signifique la aplicación de 1 2 metro v 2 a un valor conocido de metro y un valor medido de v . Ese valor medido ciertamente dependería del marco. Pretendo que la "energía cinética" sea una cantidad de energía cinética que se puede determinar sin la medición de v , sino observando la transferencia de una cantidad conocida de energía no cinética a energía cinética. Muéstrame cualquier pregunta relacionada que discuta lo último y eliminaré mi pregunta.
@sammygerbil Mientras tanto, reformularé la pregunta.
He revisado extensamente el encabezado y el cuerpo de la pregunta. Quería eliminar el original y publicar uno nuevo, pero el sitio reaccionó con un mensaje advirtiendo contra la eliminación.
@D.Ennis ¡Gracias por la aclaración! Esta pregunta ahora es mucho mejor que antes; Agregué una respuesta.

Respuestas (4)

Hay varios errores conceptuales.

  • Se supone que el trabajo realizado es el mismo en ambos marcos. Pero el trabajo también es relativo. Una forma de ver esto es notar que
    W = F v d t .
    Aunque sus dos observadores están de acuerdo en F , no están de acuerdo en v , por lo que no estarán de acuerdo en W . En particular, en el segundo cuadro, el resorte realiza un trabajo negativo sobre el bloque.
  • Se supone que el cambio en la energía potencial elástica es igual al cambio en la energía cinética del bloque. Esto es incorrecto, porque tienes que tener en cuenta el cambio en la energía cinética de lo que sea que esté en el otro extremo del resorte.
  • Una de las respuestas afirma que la cantidad de energía almacenada en un resorte es relativa. Esto es incorrecto, y usted tiene razón.

Si nos deshacemos de estos problemas, la respuesta sale bien. Digamos que el bloque tiene masa metro , y está unido a un gran objeto de masa METRO metro en el otro extremo del resorte. Considere su segundo observador. En su marco, la energía inicial es

1 2 metro v 2 + 1 2 METRO v 2 .
A continuación, dejamos que el resorte se extienda. La velocidad del bloque cambia de v a 0 . Por conservación de la cantidad de movimiento, la velocidad del objeto grande cambia de v a v + ( metro / METRO ) v . La energía final es
1 2 METRO ( v + ( metro / METRO ) v ) 2 1 2 METRO v 2 + metro v 2 .
Es decir, la energía ha aumentado en metro v 2 / 2 , como esperamos. Esta es exactamente la energía potencial elástica almacenada inicialmente en el resorte, como calculaste en el marco del primer observador.

Elegante y lúcido. Pero sugiero que Q, la energía almacenada en el resorte, se agregue a la primera ecuación. El observador en movimiento ve eso como parte de la energía inicial en el sistema. Luego, la última oración podría ser más explícita sobre el hecho de que tanto la energía elástica como la energía cinética del bloque fueron a cero, mientras que la EC de la masa más grande, M, aumentó en su suma. Todo está ahí ahora, pero el lector tiene que desenterrarlo.

Creo que Lucas tiene razón.

Debemos recordar que, en el marco en movimiento, el resorte y el bloque comprimidos (antes de que se suelte el resorte) también se observarán en movimiento, por lo que ya tendrán energía cinética y potencial.

Esta energía cinética original siempre se puede restar de la energía cinética total medida después de soltar el resorte.

La cantidad resultante de energía cinética debe ser idéntica a la medida por el observador estacionario.

( k mi s pag r i norte gramo + b yo o C k ) b mi F o r mi + ( PAG mi s pag r i norte gramo + b yo o C k ) b mi F o r mi = ( k mi s pag r i norte gramo ) a F t mi r + ( k mi b yo o C k ) a F t mi r

Entonces, aunque se puede ver que el bloque se mueve más lento o más rápido en el marco en movimiento que en el marco estacionario, de hecho es el movimiento mismo del marco el que debería corregir la "energía cinética faltante".

Se puede demostrar que este mismo argumento (con algunas modificaciones básicas) es cierto incluso si los efectos relativistas son significativos.

Creo que está haciendo un punto falso, basado en una ambigüedad en la definición de la ley de Hooke:

F = k X

significa que tomas X   como el cambio desde la posición de reposo X 0   de la primavera En un sistema en el que el resorte está en reposo, este cambio es el mismo que el s   en la formula de trabajo

W = F s = 1 2 k X 2

Ahora, cuando calcula el trabajo realizado en otro sistema de referencia, no puede usar esta fórmula. En su ejemplo, si un observador se mueve hacia el bloque (digamos de izquierda a derecha), verá que se mueve más lento, por lo que medirá su KE en el punto medio como menor que 1 2 k X 2 , pero el bloque, en su sistema de referencia, se ha movido hacia la izquierda algún tiempo después de que se soltara el resorte (su v 0 < 0 en ese marco de referencia), por lo que el trabajo realizado por el resorte en ese sistema de referencia es negativo hasta que la velocidad del bloque se vuelve positiva, es decir, hasta que la velocidad del bloque en el sistema de referencia estático excede la velocidad del sistema de referencia en movimiento. Por lo tanto, no sorprende que su KE sea inferior: en el marco de referencia en movimiento, el resorte realizó menos trabajo en el bloque. Lo que sigue siendo cierto es que Δ k = L : para ver esto, basta con tomar el caso en el que   v R = 1 2 v METRO A X . Entonces   v 0 = v METRO A X   y Δ k = 0 , pero como se puede ver en la integral, L = 0 también. (Estoy usando el apóstrofe para el marco de referencia en movimiento)

En el mejor de los casos, está mostrando que el observador en movimiento calcularía una cantidad diferente de energía liberada, según la observación del movimiento del resorte, que el observador estacionario. Sin embargo, no hicieron eso. Acordaron a priori que se almacenaban y liberaban Q julios. Podría haber usado un clavo de hierro disparado desde un solenoide en lugar del bloque y el resorte.
D.Ennis estás usando la fórmula tu = 1 2 k X 2 . Esto es correcto solo en el marco de referencia donde el resorte está estacionario.
Quizás nuestros comentarios se cruzaron en el correo.
No entiendo lo que quieres decir. Mi opinión sobre tu pregunta es que se basa en la premisa falsa de que en diferentes marcos de referencia la energía almacenada es la misma: puedes usar la fórmula tu = 1 2 k X 2 sólo en el marco estático. D.ennis, la energía potencial y la energía cinética no son invariantes, sus variaciones sí lo son.
Lo intentaré una vez más. Ambos observadores están de acuerdo en que k es k, x² es x² y, por lo tanto, que la energía almacenada en el resorte es ½kx², que es Q julios. Si lo desea, imagine que comprimieron el resorte en el marco estacionario y luego uno de ellos entró en el marco móvil.
No. En el marco en movimiento no puedes usar la fórmula. tu = 1 2 k X 2 . Haz la integral en los dos casos y compruébalo por ti mismo. Despedida

de acuerdo con el marco que se mueve con velocidad v, tanto el bloque como el resorte se movían con velocidad v en dirección opuesta cuando la fuerza del resorte actúa sobre el bloque, actúa en dirección opuesta a la dirección de la velocidad relativa del bloque con respecto al marco que se mueve con velocidad v. Ahora en el marco de referencia, es decir, el marco que se mueve con velocidad v, el bloque se mueve en dirección opuesta a la fuerza de resorte conservadora que actúa sobre él y cuando finalmente se detiene, su KE se ha convertido en PE. Ahora, en lo que respecta a su energía Q dada. En comparación con el marco estacionario, es 1/2kx ^ 2, ya que el bloque se mueve en la dirección de la fuerza del resorte, pero en comparación con el marco moviéndose con velocidad v it -Q, ya que ahora el trabajo realizado por el resorte en el bloque es -1/2kx ^ 2.Para ser más precisos, en el marco estacionario, la energía potencial inicial es Q, que se transforma en energía cinética del bloque y en el marco que se mueve con velocidad v, la energía potencial inicial del resorte es Q y la energía cinética del bloque es 1/2mv^2, esta energía cinética es cambiando a energía potencial del resorte como trabajo realizado por la fuerza del resorte, es decir, la fuerza conservativa es negativa en este marco.